Tô màu miền

Một phần của tài liệu (LUẬN văn THẠC sĩ) hình học tổ hợp với các phương pháp chứng minh luận văn ths toán học 84601 (Trang 74 - 76)

3.4 Phép tô màu

3.4.2 Tô màu miền

Bài 3.4.4. Chứng minh rằng nếu mặt phẳng bị chia bởi n đường thẳng thành nhiều miền, thì mỗi miền có thể được tô bằng một trong hai màu sao cho hai miền bất kỳ có cùng màu khơng có nhiều hơn một điểm chung.

Giải. Ta chứng minh khẳng định bằng phép quy nạp với n. Với n = 1 rõ ràng ta chỉ cần tô màu khác nhau cho hai nửa mặt phẳng. Ta giả sử rằng khẳng định đúng với mọi n = k ≥ 1, và xét hệ P gồmk+1đường bất kỳ trong mặt phẳng. Ta chọn một đường bất kỳ l ∈ P và sau đó xét một cách tơ màu miền thích hợp mà mặt phẳng bị chia bởi hệ k đường

Hình 3.32

một hoặc một nửa mặt phẳng cịn lại được xác định bởi đường l, trong khi phần còn lại bị cắt thành hai miền mới bởi l. Những cặp miền mới này là lý do tại sao cho phép tô màu cho hệP0 phải được chuyển thành tô màu cho hệP. Ta làm như sau: Trong một nửa mặt phẳng bị giới hạn bởil (phần trên của hình 3.32a) ta giữ ngun phép tơ màu, trong khi ta thay đổi màu của mỗi miền trong nửa mặt phẳng cịn lại (hình 3.32b). Do đó, ta thu được cách tơ màu có thể đối với hệ P, vì các cặp miền lân cận cũng có màu khác nhau trong trường hợp có đường biên chung (một đoạn thẳng có chiều dài dương, một đường hoặc một nửa đường)

là một phần của đườngl. Chứng minh hoàn thành.

Bài 3.4.5. Mặt phẳng bị chia thành nhiều miền bởi n đường thẳng cho trước

(n ≥ 3). Giả sử một số miền được tô màu (với một màu) sao cho hai miền bất kỳ được tơ màu có khơng nhiều hơn một điểm biên chung. Chứng minh rằng số miền được tô màu không vượt quá n23+n.

Giải. Ký hiệu p là số miền được tơ màu, với p = |S1| +|S2|, trong đó S1, S2 lần lượt là lớp các miền bị chặn, tương ứng không bị chặn, được tô màu. Bất đẳng thức p ≤ n23+n rõ ràng đúng trong trường hợp hệ các đường bao gồmnđường song song, trong trường hợp này ta cóS1 = ∅

và |S2| ≤ n+1, và hơn nữa, bất đẳng thức n+1 ≤ n23+n đúng với mọi n ≥ 3. Trong các trường hợp cịn lại, ta có được cận mong muốn bằng cách cộng các bất đẳng thức

|S1| ≤ n(n−2)

3 và|S2| ≤ n, (3.16)

mà bây giờ ta sẽ chứng minh. Lưu ý rằng mỗi một trongnđường thẳng của hệ được cắt thành hai nửa đường thẳng và tối đa(n−2)đoạn thẳng

(chiều dài dương) bởi giao điểm với(n−1) đường khác. Tính tất cả, ta có được một hệ2nnửa đường thẳng và tối đa n(n−2) đoạn thẳng, tạo thành biên của tất cả các miền, trong đó mỗi nửa đường thẳng và đoạn thẳng nằm ở biên của nhiều nhất một miền được tơ màu. Vì mỗi miền

M ∈ S1 là một đa giác k đỉnh với mọi k ≥ 3, ta có 3|S1| ≤ n(n−2), dẫn đến bất đẳng thức đầu tiên trong (3.16). Biên của mỗi miềnM ∈ S2

gồm hai nửa đường thẳng, và do đó ta có2|S2| ≤ 2n, dẫn đến bất đẳng

thức thứ hai trong (3.16). Hoàn thành chứng minh.

Một phần của tài liệu (LUẬN văn THẠC sĩ) hình học tổ hợp với các phương pháp chứng minh luận văn ths toán học 84601 (Trang 74 - 76)

Tải bản đầy đủ (PDF)

(90 trang)